Line 5: Line 5:
  
 
= [[ECE_PhD_Qualifying_Exams|ECE Ph.D. Qualifying Exam]]: Automatic Control (AC)- Question 1, August 2007=
 
= [[ECE_PhD_Qualifying_Exams|ECE Ph.D. Qualifying Exam]]: Automatic Control (AC)- Question 1, August 2007=
X and Y are iid random variable with
 
 
<math> P(X=i) = P(Y=i) = \frac {1}{2^i}\ ,i = 1,2,3,... </math>
 
 
a) Find <math> P(min(X,Y)=k)\ </math>.
 
 
b) Find <math> P(X=Y)\ </math>.
 
 
c) Find <math> P(Y>X)\ </math>.
 
 
d) Find <math> P(Y=kX)\ </math>.
 
 
----
 
----
=Solution 1 (retrived from [[Automatic_Controls:Linear_Systems_(HKNQE_August_2007)_Problem_1|here]])=
+
==Question==
 
+
Write question here
*To find <math> P(min(X,Y)=k)\ </math>, let  <math> Z = min(X,Y)\ </math>. Then finding the pmf of Z uses the fact that X and Y are iid
+
----
        <math> P(Z=k) = P(X \ge k,Y \ge k) = P(X \ge k)P(Y \ge k) = P(X \ge k)^2 </math>
+
=Solution 1=
 
+
write it here
        <math> P(Z=k) = \left ( \sum_{i=k}^N \frac {1}{2^i} \right )^2 = \left ( \frac {1}{2^k} \right )^2 = \frac {1}{4^k} </math>
+
 
+
*To find <math> P(X=Y)\ </math>, note that  X and Y are iid and summing across all possible i,
+
        <math> P(X=Y) = \sum_{i=1}^\infty P(X=i, Y=i) = \sum_{i=1}^\infty P(X=i)P(Y=i) = \sum_{i=1}^\infty \frac {1}{4^i} = \frac {1}{3} </math>
+
 
+
*To find <math> P(Y>X)\ </math>, again note that X and Y are iid and summing across all possible i,
+
        <math> P(Y>X) = \sum_{i=1}^\infty P(Y>i, X=i) = \sum_{i=1}^\infty P(Y>i)P(x=i)</math>
+
 
+
:Next, find <math> P(Y<i)\ </math>
+
        <math> P(Y>i) = 1 - P(Y \le i) </math>
+
 
+
        <math> P(Y \le i) = \sum_{i=1}^\infty \frac {1}{2^i} = 1 + \frac {1}{2^i} </math>
+
 
+
      <math> \therefore P(Y>i) = \frac {1}{2^i} </math>
+
 
+
:Plugging this result back into the original expression yields
+
        <math> P(Y<X) = \sum_{i=1}^\infty \frac {1}{4^i} = \frac {1}{3} </math>
+
 
+
 
+
*To find <math> P(Y=kX)\ </math>, note that X and Y are iid and summing over all possible combinations one arrives at
+
        <math> P(Y=kX) = \sum_{i=1}^\infty i = 1^\infty P(Y=ki, X=i) = \sum_{i=1}^\infty P(Y=ki)P(X=i) </math>
+
:Thus,
+
        <math> P(Y=kX) = \sum_{i=1}^\infty \frac {1}{2^{ki}} \frac {1}{2^i} = \sum_{i=1}^\infty \frac {1}{2^{(k+1)i}} = \frac {1}{2^{(k+1)}-1} </math>
+
 
----
 
----
 
==Solution 2==
 
==Solution 2==

Revision as of 08:07, 27 June 2012


ECE Ph.D. Qualifying Exam: Automatic Control (AC)- Question 1, August 2007


Question

Write question here


Solution 1

write it here


Solution 2

Write it here.


Back to ECE Qualifying Exams (QE) page

Alumni Liaison

Ph.D. 2007, working on developing cool imaging technologies for digital cameras, camera phones, and video surveillance cameras.

Buyue Zhang